please help!!! urgent

Please Help!!! Urgent

Answers

Answer 1

Answer:

E

Step-by-step explanation:

is the answer


Related Questions

Order the numbers from least to greatest.
1.3 ,-2, 1.8 , 0, -1.75

Answers

Answer:

-2 , -1.75 , 0 ,1.3 , 1.8

Step-by-step explanation:

[tex]1.3 ,\:-2,\: 1.8 , \:0,\: -1.75[/tex] =

[tex]-2 , -1.75 \:, 0\: ,1.3 \:, 1.8[/tex]

A negative number is a always smaller than a positive number.

The smallest negative number among the given set of data is ; -2.

The largest positive number is 1.8

Please answer this question

Answers

Answer:

I think it would be B but im not completly sure.

Convert 2.42 to percent

Answers

Answer:

242%

Step-by-step explanation:

Answer:

242%

Step-by-step explanation:

Whole numbers are a 100% each, so there's 200%. Tenths are 10% every tenth, so 40%, and hundredths are 1% every hundredth, so 2%.

200+40+2=242%

If it was helpful, could you plz consider giving me brainliest?

what is 360 x 4500 plsssssssssssssssss help

Answers

Answer:

1,620,000

Step-by-step explanation:

360×4500

=1,620,000

is the answer

Hey there!

[tex]Answer:\boxed{1,620,000}[/tex]

[tex]Explanation:[/tex]

[tex]360*4500[/tex]

Let's multiply to get our answer.

[tex]360*4500=1,620,000[/tex]

[tex]1,620,000[/tex]

Hope this helps!

Determine whether the given segments have the same length. Complete the justification of your answer,

AB and BC


The length of AB is

length

The length of BC is /

.So, AB and BC (select)

the same length

Answers

Answer:

No does not contain the same length

Step-by-step explanation:

Given that

A(-4,2) , B (1,4), C (2, -1)

Now

AB would be

= [tex]\sqrt{(1-(-4))^2 + (4-2)^2} \\\\= \sqrt{29}[/tex]

And, BC is

[tex]= \sqrt{(2-1)^2 + 1-(-4))^2} \\\\= \sqrt{26}[/tex]

As we can see that the AB and BC are not equaled

So,

AB ≠ BC

Due to the different length

Therefore they do not contains the same length

We simply applied the distance formula due to which the length could be determined and the same is to be considered

Last week, Lindsay earned $10 per hour plus a $60 bonus for good job performance. She spends StartFraction 1 Over 15 EndFractionof her paycheck on dinner with friends. If she had not earned the bonus, the amount she spent on dinner would have been StartFraction 1 Over 10 EndFraction of her paycheck. Which equation can be used to find h, the number of hours Lindsay worked last week? StartFraction 1 Over 15 EndFraction left-parenthesis 10 h plus 60 right-parenthesis equals StartFraction 1 Over 10 Endfraction left-parenthesis 10 h right-parenthesis(10h + 60) = StartFraction 1 Over 10 EndFraction(10h) StartFraction 1 Over 15 EndFraction(10h + 60h) = StartFraction 1 Over 15 EndFraction left-parenthesis 10 h plus 60 h right-parenthesis equals StartFraction 1 Over 10 Endfraction left-parenthesis 10 h right-parenthesis(10h) StartFraction 1 Over 15 EndFractionh(10 + 60) = StartFraction 1 Over 15 EndFraction h left-parenthesis 10 plus 60 right-parenthesis equals StartFraction 1 Over 10 Endfraction h left-parenthesis 10 right-parenthesish(10) StartFraction 1 Over 15 EndFraction left-parenthesis 10 plus 60 h right-parenthesis equals StartFraction 1 Over 10 Endfraction left-parenthesis 10 h right-parenthesis(10 + 60h) = StartFraction 1 Over 10 EndFraction(10h)

Answers

Answer:

StartFraction 1 Over 15 EndFraction left-parenthesis 10 h plus 60 right-parenthesis equals StartFraction 1 Over 10 Endfraction left-parenthesis 10 h right-parenthesis

Step-by-step explanation:

The desired equation equates 1/15 of the amount Lindsay earned with 1/10 the amount without the bonus:

StartFraction 1 Over 15 EndFraction left-parenthesis 10 h plus 60 right-parenthesis equals StartFraction 1 Over 10 Endfraction left-parenthesis 10 h right-parenthesis

_____

Comment on the form of the answer

If you don't like answers in this form, please do not post questions in this form. Ordinary math symbols are much appreciated.

Answer:

A

Step-by-step explanation:

edge 23

PLEASEPLEASEHELP i need

Answers

Answer:

B

Step-by-step explanation:

hope you get the best grade

Write a system of inequalities to match the graph and prove that the coordinate point (-10, -14) is in the solution set

Answers

Answer:

y < x - 2

y ≥ 2x + 5

Step-by-step explanation:

For the equation of the dotted line,

Slope of the line = [tex]\frac{y_2-y_1}{x_2-x_1}[/tex]

Since the dotted lines passes through the points (-7, -9) and (2, 0)

Slope = [tex]\frac{-9-0}{-7-2}[/tex]

    [tex]m_1[/tex] = 1

Therefore, equation of the line passing through (2, 0) and slope = 1 will be,

y - y' = m₁(x - x')

y - 0 = 1(x - 2)

y = x - 2

Since, shaded area is below the dotted line,

y < x - 2 will be the inequality.

Slope of the solid line passing through (-7, -9) and (0, 5) is,

[tex]m_2=\frac{-9-5}{-7-0}[/tex]

     = 2

Therefore, equation of the line passing through (0, 5) and slope = 2 will be,

y - 5 = 2(x - 0)

y = 2x + 5

Since, the given line is a solid line and shaded area is above the line,

y ≥ 2x + 5 will be the inequality.

Since, (-10, -14) is lying in the common shaded area of both the inequalities, it will be the solution set of the graphed inequalities.

34=9r-11
help plzzzzzzzzzzzzz hurry

Answers

Answer:

5

Step-by-step explanation:

Swap sides so that all variable terms are on the left hand side.

9r−11=34

Add 11 to both sides.

9r=34+11

Add 34 and 11 to get 45.

9r=45

Divide both sides by 9.

r= 45/9

Divide 45 by 9 to get 5.

r=5

The answer is 5
Move -12 to the left and add it to 34 which gets 45
Divide 9 from 45 and you get 5

70,000 inches equals how many miles

Answers

Answer:

1.104798 miles

odds against a randomly chosen household having a cat if the probability of the event that a randomly chosen household has a cat is .21

Answers

Step-by-step explanation:

Odds against = failures / successes

Probability = successes / (successes + failures)

0.21 = cats / (cats + no cats)

1 / 0.21 = (cats + no cats) / cats

4.76 = 1 + (no cats / cats)

3.76 = no cats / cats

So the odds against are 3.76 to 1.  

Suppose PR = 54, solve for QR
4x-1
3x-1
P
R

Answers

Answer:

[tex] QR = 23 [/tex]

Step-by-step explanation:

P, A, and R are collinear.

PR = 54

[tex] PQ = 4x - 1 [/tex]

[tex] QR = 3x - 1 [/tex]

To solve for the numerical length of PR, let's generate an equation to find the value of x.

According to the segment addition postulate:

[tex] PQ + QR = PR [/tex]

[tex] (4x - 1) + (3x - 1) = 54 [/tex] (substitution)

Solve for x

[tex] 4x - 1 + 3x - 1 = 54 [/tex]

Combine like terms

[tex] 4x + 3x - 1 - 1 = 54 [/tex]

[tex] 7x - 2 = 54 [/tex]

Add 2 to both sides

[tex] 7x - 2 + 2 = 54 + 2 [/tex]

[tex] 7x = 56 [/tex]

Divide both sides by 7

[tex] \frac{7x}{7} = \frac{56}{7} [/tex]

[tex] x = 8 [/tex]

[tex] QR = 3x - 1 [/tex]

Plug in the value of x into the equation

[tex] QR = 3(8) - 1 = 24 - 1 [/tex]

[tex] QR = 23 [/tex]

For which values of a the system has at least one solution: 1.x≤7 ,x≥a

Answers

Given:

The system of inequalities

[tex]x\leq 7[/tex]

[tex]x\geq a[/tex]

To find:

The values of a for which the system has at least one solution.

Solution:

We have,

[tex]x\leq 7[/tex]     ...(i)

It means all real values of x which are less than or equal to 0.

[tex]x\geq a[/tex]       ...(ii)

It means all real values of x which are greater than or equal to a.

Using (i) and (ii), we get

[tex]a\leq x\leq 7[/tex]

The value of x is lies in the interval [a,7].

It means, if the value of a is less than or equal to 7, then system has at least one solution.

But if a is more than 7, then the system of inequalities have to solution.

Therefore, the value of a must be less than or equal to 7.

Liam bought pizza and wings for 27.58 how much change should liam receive if he gave the clerk three $10 bills

Answers

Answer:

$2.52

Step-by-step explanation:

i did it

On a test, Avery lost 5
points for each of the 6
questions she got wrong
but earned 10 points for
answering the bonus
question correctly. What
integer represents her
score in relation to a
perfect score of 100?

Answers

Answer:

bruhhhhhhhhhhhhhhhhhhhhhhhhhh

Answer these questions please

Answers

Step-by-step explanation:

hope This helps for you

Answer:

Step-by-step explanation:

Common factors are the number's , variables found in each of the terms

1) 21x²y = 3 * 7 * x * x * y

  35xy² = 5 * 7 * x    *  y * y

Common factor = 7 * x * y

                          = 7xy

Answer :(iii) 7xy

2)a² - b² = (a + b)(a - b)

1 - x² = (1 - x) (1 +x)

(ii)

3) x³ - x = x(x² - 1)

            = x(x + 1)(x - 1)

Answer: (iv)

4) 225 =  15 * 15 = 15²

x³ - 225x = x(x² - 225)

                = x(x² - 15²)            {a² - b² =(a + b)(a - b)}

                = x(x + 15)(x - 15)

Answer: (ii)

5) 12a²b = 3 * 4 * a * a * b

   15ab² = 3 * 5 * a *     b * b

Common factor = 3 * a * b = 3ab

12a²b + 15ab² = 3*4*a²*b +  3*5*a*b²

                      = 3ab (4a + 5b)

Answer:(ii)

6. Convert 23% to a fraction and then
a decimal.

Answers

Answer:

23/100

0.23

Step-by-step explanation:

Hope it helps....

Answer:

23% as a fraction is 23/100.

23% as a decimal is 0.23

Step-by-step explanation:

tristan bought an orange that weighs 1 8th of a pound

Answers

Answer

Tristan bought an orange that weighs 1 8th of a pound. 1. See answer. Add answer. Log in to add comment.

Arturo’s gym membership costs $100 per year plus $20 per month. Which equation represents this relationship?

Answers

Answer:

$100+$20=$120

Can somebody help and crAck the code this is 100 points if you get it right your are gonna be the brainliest

Answers

Answer:

RECORD TIME

Step-by-step explanation:

RECORD TIME

Answer:

record time

Step-by-step explanation:

░░░░░░░░███████████████░░░░░░░░

░░░░░█████████████████████░░░░░

░░░░████████████████████████░░░

░░░██████████████████████████░░

░░█████████████████████████████

░░███████████▀░░░░░░░░░████████

░░███████████░░░░░░░░░░░░░░░███

░████████████░░░░░░░░░░░░░░░░██

░█░░███████░░░░░░░░░░░▄▄░░░░░██

█░░░░█████░░░░░░▄███████░░██░░█

█░░█░░░███░░░░░██▀▀░░░░░░░░██░█

█░░░█░░░░░░░░░░░░▄██▄░░░░░░░███

█░░▄█░░░░░░░░░░░░░░░░░░█▀▀█▄░██

█░░░░░░░░░░░░░░░░░░░░░░█░░░░██░

░███░░░░░░░░░░░░░░░░░░░█░░░░█░░

░░█░█░░░░░░░█░░░░░██▀▄░▄██░░░█░

░░█░█░░░░░░█░░░░░░░░░░░░░░░░░█░

░░░██░░░░░░█░░░░▄▄▄▄▄▄░░░░░░█░░

░░░██░░░░░░░█░░█▄▄▄▄░▀▀██░░█░░░

░░░██░░░░░░░█░░▀████████░░█░░░░

░░█░░█░░░░░░░█░░▀▄▄▄▄██░░█░░░░░

░░█░░░█░░░░░░░█░░░░░░░░░█░░░░░░

░█░░░░░█░░░░░░░░░░░░░░░░█░░░░░░

░░░░░░░░█░░░░░░█░░░░░░░░█░░░░░░

░░░░░░░░░░░░░░░░████████░░░░░░░

Certain investments compound interest at different intervals. What effect does the size of the compounding interval have
on the yield of the investment?
a. Investments with smaller compounding intervals have a higher yield, because the interest
earned is reinvested more quickly and thus gains compound interest more quickly.
b. Investments with smaller compounding intervals have a lower yield, because the interest earned
is smaller at each interval.
c. The size of the compounding interval does not affect the total yearly interest percentage rate and
only exists for bookkeeping purposes.
d. Investments with smaller compounding intervals have a lower yield, because the bank charges
a small fee every time interest is compounded.

Answers

Answer:

Option A: Investments with smaller compounding intervals have a higher yield, because the interest earned is reinvested more quickly and thus gains compound interest more quickly

Step-by-step explanation:

Compounding could be done monthly, quarterly, semiannually or annually. Compound interest is simply when we add the interest to the principal sum of a loan or amount deposited. It's also referred to as interest which is accrued on interest.

The compound interest that will be earned mainly depends on the frequency of compounding. Now compounding could be done monthly, quarterly, biannually or annually. Now, If we compound monthly, we will have a higher interest return than if we compounded annually.

Thus, looking at the options, the correct one is Option A

Answer:

a

Step-by-step explanation:

• She used a coupon for $3.00 off the total price of the items she
This expression can be used to determine the price of the items Nakit
(2(3.50 -0.80) + 4.85 + 2.40) – 3.00
What is the price of the items Nakita bought?
A $6.95
B $10.45
C $9.65
D $12.65

Answers

The answer is c 9.65

which systems of equations have no Solutions ​

Answers

When both sides don’t equal each other

Brainliest?

Answer:

Inconsistent System of Equations

Explanation:

There is no solution for the system of equations that graphs as parallel lines.

Parallel lines never cross, therefore there is no intersection. This is an inconsistent system of equations.

Find the equation of the line through (10, 4) that is parallel to the line with the equation
y = -1x + 3. Give the equation in standard form.

Answers

Answer:

  x + y = 14

Step-by-step explanation:

The question asks for the final answer to be an equation in standard form. The standard form of the equation for a line is ...

  ax +by = c

where a, b, c are integers with no common factors, and 'a' is positive.

__

When working with parallel and perpendicular lines, standard form is an easy form to work with. The coefficients (a and b) of parallel lines will be the same. All that is different between parallel lines is the value of 'c'.

Given this fact, it is convenient to start by putting the given equation into standard form. For that, we want the x-term on the same side of the equal sign as the y-term.

Adding x to both sides of the given equation puts it in standard form:

  x +y = x +(-x) +3

  x + y = 3 . . . . . simplified

__

Now that we know our equation will be of the form ...

  x + y = (some constant)

we need to find the constant. We can do that by using the given point values for x and y.

For the given point (x, y) = (10, 4), the constant for the parallel line will be ...

  x + y = 10 + 4 = 14

  x + y = 14 . . . equation of parallel line through (10, 4)

Determine the largest integer value of x in the solution of the following inequality. -3x +1> 7

Answers

Answer:

x<1/3

Step-by-step explanation:

An unbiased sample is one in which every member of the group has an equal chance of being chosen.
True
False

Answers

This is true, because everyone has a fair chance
True, biased is pushing towards one side

Find the surface area of a square pyramid if the area of the base is 324 cm^2 and the height is 40 cm.

Answers

Answer:

Step-by-step explanation:

Yep!

Which statement is true for the graph of f(x) = 2x2 - 6x2 - 48x + 24?
(Answers attached)

Answers

Answer:

min = (4, -136) ; max = (-2,80)

Step-by-step explanation:

The option is B

The (-2, 80) is a relative minimum, (4, -136) relative maximum option (C) is correct.

What is a function?

It is defined as a special type of relationship, and they have a predefined domain and range according to the function every value in the domain is related to exactly one value in the range.

It is given that:

The cubic function:

f(x) = 2x³ - 6x² - 48x + 24

Find f'(x):

f'(x) = 6x² - 12x - 48

Equate f'(x) to zero:

6x² - 12x - 48 = 0

Find the roots of the quadratic equation:

x = -2

x = 4

Find f''(x):

f''(x) = 12x - 12

Plug x = -2

f''(-2) = -24 - 12 = -36 < 0

At x = -2 f(x) is minimum

f(-2) = 80

Plug x = 4

f''(4) = 48 - 12 = 36 > 0

At x = 4 f(x) is maximum

f(4) = -136

Thus, the (-2, 80) is a relative minimum, and (4, -136) relative maximum option (C) is correct.

Learn more about the function here:

brainly.com/question/5245372

#SPJ2

Please help!!!!!!!!!!!!!

Answers

Answer:

r=8

Step-by-step explanation:

r*4=32

32/4=8

Therefore, r=8

Answer:

r = 8

Step-by-step explanation:

Ok don't worry. This question is not as hard as it should be.

It states it's a balanced equation. So that means 32 is equal to the other side.

there are 4 r's on the other side

32=4r

Divide 4 on both sides so the variable is independent

32/4=4r/4

8=r

Hope this helps!

Find the distance between the two points (5, -3) and (2, 5). Simplify your answer, and write the exact answer in simplest radical form for an irrational answer.

Answers

Answer:

(3.5, 1)

Step-by-step explanation:

Other Questions
William and his puppy Rover are out for a morning jog at a comfortable pace of 10 minutes per mile. If they keep up this pace for 2 hours, how many miles will they jog? Find the sum and then classify it. StartFraction 5 over 6 EndFraction + StartRoot 91 EndRoot 9.58297100, irrational 10.ModifyingAbove 3 with bar, rational 10.3727253, irrational 91.8ModifyingAbove 3 with bar, rational Which type of magma is known for the most violent eruptions? Basaltic Rhyolitic Andesitic Both Basaltic and Andesitic If each group has the greatest possible number of club members,how many groups of sixth graders and how many groups of seventhgraders will there be? Use numbers and words to explain your answer. How is the theme of "reality can be much different than appearances" developed in "The Necklace"? Select the two correct answers. 1.Mathilde ages after years of working hard to pay for the lost necklace. 2.Mathilde's husband is able to obtain an invitation to the ball, even though he is not rich. 3.Mathilde is not rich, but she appears to be so when she borrows the necklace. 4.The necklace Mathilde thinks is made of diamonds turns out to be made of paste. y < 2x - 5 Which of the following graphs represent the inequality? What is the value of m Determine if the statement is true or false.An anonymous tip can provide probable cause for a search warrant. Convert 50 grains to milligrams which diagram show lines that must be parallel lines cut by a transversal Comprehensive CVP analysis "Ill never understand this accounting stuff," Blake Dunn yelled, waving the income statement he had just received from his accountant in the morning mail. "Last month, we sold 2,000 stuffed State University mascots and earned $6,565 in operating income. This month, when we sold 3,000 I thought wed make $9,848. But this income statement shows an operating income of $11,615! How can I ever make plans if I cant predict my income? Im going to give Janice one last chance to explain this to me," he declared as he picked up the phone to call Janice Miller, his accountant. What is the value of the expression below?435+3.550170 18110219024 explain why a cell a matter? PLZ HELP I WILL MARK YOU BRAINLIST Order the following rational numbers from greatest to smallest.- 70.404.1-3/4 maria y nora ___ trabajadoras.i have no clue Silus wants to monitor the temperature of a reaction every 0.5 seconds for 30 minutes. He plans to generate a graph of the temperature values over time and insert the graph into a text document. Which pair of tools would be best for Silus to use? The sum of two numbers is 4 and the difference is -6. What is the product of the two numbers? Which expression can be used to solve3/5| 7/10 Ais a changeable factor in an experiment.O THEORYO LAWO HYPOTHESISO CONTROLO VARIABLE Let X1,..., Xn be a simple random sample from a distribution with density function Svorvo-1 O 0 is an unknown parameter. (a) Find a MOM estimator for 0. (b) If the observations are 1 1 1 2'3'2 Determine the point estimate with the estimator you find in part (a).